Difference between revisions of "2013 AIME II Problems/Problem 13"
Mathgeek2006 (talk | contribs) (→Solution) |
|||
Line 2: | Line 2: | ||
In <math>\triangle ABC</math>, <math>AC = BC</math>, and point <math>D</math> is on <math>\overline{BC}</math> so that <math>CD = 3\cdot BD</math>. Let <math>E</math> be the midpoint of <math>\overline{AD}</math>. Given that <math>CE = \sqrt{7}</math> and <math>BE = 3</math>, the area of <math>\triangle ABC</math> can be expressed in the form <math>m\sqrt{n}</math>, where <math>m</math> and <math>n</math> are positive integers and <math>n</math> is not divisible by the square of any prime. Find <math>m+n</math>. | In <math>\triangle ABC</math>, <math>AC = BC</math>, and point <math>D</math> is on <math>\overline{BC}</math> so that <math>CD = 3\cdot BD</math>. Let <math>E</math> be the midpoint of <math>\overline{AD}</math>. Given that <math>CE = \sqrt{7}</math> and <math>BE = 3</math>, the area of <math>\triangle ABC</math> can be expressed in the form <math>m\sqrt{n}</math>, where <math>m</math> and <math>n</math> are positive integers and <math>n</math> is not divisible by the square of any prime. Find <math>m+n</math>. | ||
==Solution== | ==Solution== | ||
− | After drawing the figure, we suppose <math>BD=a</math>, so that<math>CD=3a</math>,<math>AC=4a</math>, and <math>AE=ED=b</math>. | + | |
+ | === Solution 1 === | ||
+ | After drawing the figure, we suppose <math>BD=a</math>, so that <math>CD=3a</math>, <math>AC=4a</math>, and <math>AE=ED=b</math>. | ||
Using cosine law for <math>\triangle AEC</math> and <math>\triangle CED</math>,we get | Using cosine law for <math>\triangle AEC</math> and <math>\triangle CED</math>,we get | ||
− | < | + | <cmath>b^2+7-2\sqrt{7}\cdot \cos(\angle CED)=9a^2\qquad (1)</cmath> |
+ | <cmath>b^2+7+2\sqrt{7}\cdot \cos(\angle CED)=16a^2\qquad (2)</cmath> | ||
+ | So, <math>(1)+(2)</math>, we get<cmath>2b^2+14=25a^2. \qquad (3)</cmath> | ||
− | <math> | + | Using cosine law in <math>\triangle ACD</math>, we get |
− | + | <cmath>b^2+9a^2-2\cdot 2b\cdot 3a\cdot \cos(\angle ADC)=16a^2</cmath> | |
− | + | So, <cmath>\cos(\angle ADC)=\frac{7a^2-4b^2}{12ab}.\qquad (4)</cmath> | |
− | <math> | + | Using cosine law in <math>\triangle EDC</math> and <math>\triangle EDB</math>, we get |
− | + | <cmath>b^2+9a^2-2\cdot 3a\cdot b\cdot \cos(\angle ADC)=7\qquad (5)</cmath> | |
− | + | <cmath>b^2+a^2+2\cdot a\cdot b\cdot \cos(\angle ADC)=9.\qquad (6)</cmath> | |
− | <math> | + | <math>(5)+(6)</math>, and according to <math>(4)</math>, we can get |
+ | <cmath>37a^2+2b^2=48. \qquad (7)</cmath> | ||
− | <math> | + | Using <math>(3)</math> and <math>(7)</math>, we can solve <math>a=1</math> and <math>b=\frac{\sqrt{22}}{2}</math>. |
− | + | Finally, we use cosine law for <math>\triangle ADB</math>, | |
− | + | <cmath>4(\frac{\sqrt{22}}{2})^2+1+2\cdot\2(\frac{\sqrt{22}}{2})\cdot \cos(ADC)=AB^2</cmath> | |
− | + | then <math>AB=2\sqrt{7}</math>, so the height of this <math>\triangle ABC</math> is <math>\sqrt{4^2-(\sqrt{7})^2}=3</math>. | |
− | <math> | + | Then the area of <math>\triangle ABC</math> is <math>3\sqrt{7}</math>, so the answer is <math>\boxed{010}</math>. |
− | + | === Solution 2 === | |
+ | Let <math>X</math> be the foot of the altitude from <math>C</math> with other points labelled as shown below. | ||
+ | <asy> | ||
+ | size(200); | ||
+ | pair A=(0,0),B=(2*sqrt(7),0),C=(sqrt(7),3),D=(3*B+C)/4,L=C/5,M=3*B/7; | ||
+ | draw(A--B--C--cycle);draw(A--D^^B--L^^C--M); | ||
+ | label("$A$",A,SW);label("$B$",B,SE);label("$C$",C,N);label("$D$",D,NE);label("$L$",L,NW);label("$M$",M,S); | ||
+ | pair X=foot(C,A,B), Y=foot(L,A,B); | ||
+ | pair EE=D/2; | ||
+ | label("$X$",X,S);label("$E$",EE,NW);label("$Y$",Y,S); | ||
+ | draw(C--X^^L--Y,dotted); | ||
+ | draw(rightanglemark(B,X,C)^^rightanglemark(B,Y,L)); | ||
+ | </asy> | ||
+ | Now we proceed using [[mass points]]. To balance along the segment <math>BC</math>, we assign <math>B</math> a mass of <math>3</math> and <math>C</math> a mass of <math>1</math>. Therefore, <math>D</math> has a mass of <math>4</math>. As <math>E</math> is the midpoint of <math>AD</math>, we must assign <math>A</math> a mass of <math>4</math> as well. This gives <math>L</math> a mass of <math>5</math> and <math>M</math> a mass of <math>7</math>. | ||
− | + | Now let <math>AB=b</math> be the base of the triangle, and let <math>CX=h</math> be the height. Then as <math>AM:MB=3:4</math>, and as <math>AX=\frac{b}{2}</math>, we know that <cmath>MX=\frac{b}{2}-\frac{3b}{7}=\frac{b}{14}.</cmath> Also, as <math>CE:EM=7:1</math>, we know that <math>EM=\frac{1}{\sqrt{7}}</math>. Therefore, by the Pythagorean Theorem on <math>\triangle {XCM}</math>, we know that <cmath>\frac{b^2}{196}+h^2=\left(\sqrt{7}+\frac{1}{\sqrt{7}}\right)^2=\frac{64}{7}.</cmath> | |
− | + | Also, as <math>LE:BE=5:3</math>, we know that <math>BL=\frac{8}{5}\cdot 3=\frac{24}{5}</math>. Furthermore, as <math>\triangle YLA\sim \triangle XCA</math>, and as <math>AL:LC=1:4</math>, we know that <math>LY=\frac{h}{5}</math> and <math>AY=\frac{b}{10}</math>, so <math>YB=\frac{9b}{10}</math>. Therefore, by the Pythagorean Theorem on <math>\triangle BLY</math>, we get <cmath>\frac{81b^2}{100}+\frac{h^2}{25}=\frac{576}{25}.</cmath> | |
+ | Solving this system of equations yields <math>b=2\sqrt{7}</math> and <math>h=3</math>. Therefore, the area of the triangle is <math>3\sqrt{7}</math>, giving us an answer of <math>\boxed{010}</math>. | ||
==See Also== | ==See Also== | ||
{{AIME box|year=2013|n=II|num-b=12|num-a=14}} | {{AIME box|year=2013|n=II|num-b=12|num-a=14}} | ||
{{MAA Notice}} | {{MAA Notice}} |
Revision as of 03:16, 22 July 2013
Problem 13
In , , and point is on so that . Let be the midpoint of . Given that and , the area of can be expressed in the form , where and are positive integers and is not divisible by the square of any prime. Find .
Solution
Solution 1
After drawing the figure, we suppose , so that , , and .
Using cosine law for and ,we get
So, , we get
Using cosine law in , we get
So,
Using cosine law in and , we get
, and according to , we can get
Using and , we can solve and .
Finally, we use cosine law for ,
then , so the height of this is .
Then the area of is , so the answer is .
Solution 2
Let be the foot of the altitude from with other points labelled as shown below. Now we proceed using mass points. To balance along the segment , we assign a mass of and a mass of . Therefore, has a mass of . As is the midpoint of , we must assign a mass of as well. This gives a mass of and a mass of .
Now let be the base of the triangle, and let be the height. Then as , and as , we know that Also, as , we know that . Therefore, by the Pythagorean Theorem on , we know that
Also, as , we know that . Furthermore, as , and as , we know that and , so . Therefore, by the Pythagorean Theorem on , we get Solving this system of equations yields and . Therefore, the area of the triangle is , giving us an answer of .
See Also
2013 AIME II (Problems • Answer Key • Resources) | ||
Preceded by Problem 12 |
Followed by Problem 14 | |
1 • 2 • 3 • 4 • 5 • 6 • 7 • 8 • 9 • 10 • 11 • 12 • 13 • 14 • 15 | ||
All AIME Problems and Solutions |
The problems on this page are copyrighted by the Mathematical Association of America's American Mathematics Competitions.